problemsets-PS_8.pdf

download problemsets-PS_8.pdf

of 2

Transcript of problemsets-PS_8.pdf

  • 7/27/2019 problemsets-PS_8.pdf

    1/2

    KEITH DEVLIN: Introduction to Mathematical Thinking (Fall 2013) Problem Set Week 8

    This problem set focuses on material covered in Week 8 (Lecture 10), so I recommend you to watch

    the lecture and attempt Assignment 10 (both parts) before submitting your answers. The deadline for

    completing (and submitting) the problem set is Monday October 28 at 9:00AM US-PDT. Note that you

    can save your entries as you work through the problems, and can change them at any time prior to

    submission, but once you submit your answers no further changes are possible.

    1. Say which of the following are true. (Leave the box empty to indicate that its false.)

    A set A of reals can have at most one least upper bound.

    If a set A of reals has a lower bound, it has infinitely many lower bounds.

    If a set A of reals has both a lower bound and an upper bound, then it is finite.

    0 is the least upper bound of the set of negative integers, considered as a subset of the reals.

    2. Which of the following say that b is the greatest lower bound of a set A of reals? (Leave the box

    empty to indicate that it does not say that.)

    b a for all a A and if c a for all a A, then b c.

    b a for all a A and if c a for all a A, then b > c.

    b < a for all a A and if c < a for all a A, then b c.

    b < a for all a A and if c a for all a A, then b c.

    b a for all a A and if > 0 there is an a A such that a < b + .

    3. The Sandwich Theorem (also sometimes called the Squeeze Theorem) says that if{an}

    n=1, {bn}

    n=1,

    {cn}

    n=1are sequences such that, from some point n0 onwards,

    an bn cn,

    and if

    limn

    an = L , limn

    cn = L,

    then {bn}

    n=1 is convergent and

    limn

    bn = L.

    Taking the Sandwich Theorem to be correct (which it is), grade the following proof using the course

    rubric.

    Theorem limn

    sin2 n

    3n= 0

    Proof: For any n,

    0 sin

    2

    n3n 13n

    Clearly, limn

    1

    3n= 0. Hence, by the Sandwich Theorem,

    limn

    sin2 n

    3n= 0

    as required.

    1

  • 7/27/2019 problemsets-PS_8.pdf

    2/2

    4. Is the following proof of the Sandwich Theorem correct? Grade it according to the course rubric.

    Theorem (Sandwich Theorem) Suppose {an}n=1, {bn}

    n=1, {cn}n=1 are sequences such that, from

    some point n0 onwards,

    an bn cn.

    Suppose further that

    limn

    an = L , limn

    cn = L.

    Then {bn}

    n=1is convergent and

    limn

    bn = L.

    Proof: Since limn

    an = L, we can find an integer n1 such that

    n n1 |an L| <

    Since limn

    cn = L, we can find an integer n2 such that

    n n2 |cn L| <

    Let M = max{n0, n1, n2}. Then

    n M ( < anL < ) ( < cnL < )

    < anL bnL cnL < (using an bn cn)

    < bnL <

    |bn L| <

    By the definition of a limit, this proves that {bn}

    n=1is convergent and lim

    n

    bn = L, as required.

    5. Evaluate this purported proof, and grade it according to the course rubric.

    Theorem limn

    n + 1

    2n + 1 =

    1

    2 .

    Proof: Let > 0 be given. Choose N large enough so that N 1

    2.

    Then, for n N,

    n + 1

    2n + 1

    1

    2

    =

    2(n + 1) (2n + 1)

    2(2n + 1)

    =

    1

    2(2n + 1)

    =1

    2(2n + 1)